Udowodnić $\lim_{n\rightarrow\infty}\frac{n\log(n)}{\log(n!)} = 1$[duplikować]

Jan 25 2021

muszę udowodnić$$\lim_{n\rightarrow\infty}\frac{n\log(n)}{\log(n!)} = 1$$ale jestem u kresu rozumu. Przeszukałem sieć, ale mogę znaleźć tylko przykłady/odpowiedzi dowodzące odwrotności = 0 i sam wypróbowałem kilka metod bezskutecznie (terminy rozszerzone, reguła L'Hopitals z niezliczonymi różnymi wyprowadzeniami, które były błędne). Czy ktoś może wskazać mi właściwy kierunek, kompletnie utknąłem...

Odpowiedzi

3 BenjaminWang Jan 25 2021 at 08:52

Zauważ, że$\log n! = \sum_{k=1}^n \log k$. Rysując odpowiednie wykresy, możesz zobaczyć:

$$\int_1^n \log x dx \le \sum_{k=1}^n \log k $$

$$\le \int_1^{n+1} \log x dx$$

Teraz oblicz całkę$\int_1^m \log x dx = m \log m - m + 1$, więc powyższe staje się

$$n \log n - n + 1 \le \log n! \le (n+1)\log(n+1)-n$$

A teraz otrzymujemy Twój wynik z twierdzenia o ściśnięciu, po podzieleniu.

1 crystal_math Jan 25 2021 at 09:04

$\log(n!)\ge \frac{n}{2}\log(\frac{n}{2})$a więc$\dfrac{n\log(n)}{\log(n!)}\le \dfrac{n\log(n)}{\frac{n}{2}\log(\frac{n}{2})}$

Oceniając tę ​​granicę górnej granicy, otrzymasz:$2$odkąd$\lim_{n\rightarrow \infty} \dfrac{\log(n)}{\log(n/2)} = 1$. Jeśli jednak wybierzesz$\epsilon >1$, zobaczysz

$\log(n!)\ge \frac{n}{\epsilon}\log(\frac{n}{\epsilon})$a więc$$\dfrac{n\log(n)}{\log(n!)}\le \dfrac{n\log(n)}{\frac{n}{\epsilon}\log(\frac{n}{\epsilon})}\rightarrow \epsilon$$

i ponieważ$\epsilon>1$(dowolne), można wywnioskować, że$$\dfrac{n\log(n)}{\log(n!)}\le 1$$

(możesz łatwo uzyskać dolną granicę), więc granica musi być$1$.

1 zkutch Jan 25 2021 at 08:58

Za pomocą$$\left( \frac{n}{e}\right)^n \lt n! \lt e \left( \frac{n}{2}\right)^n$$mamy$$n \log \frac{n}{e} \lt \log n! \lt \log e+ n \log \frac{n}{2}$$

Dodatek.

Dla lewej strony pierwszy stopień indukcji jest jasny. Następnie$$(n+1)!=n!(n+1) \gt \left( \frac{n}{e}\right)^n (n+1) = \\ =\left( \frac{n+1}{e}\right)^{n+1} \frac{(n+1)\left( \frac{n}{e}\right)^n}{\left( \frac{n+1}{e}\right)^{n+1}} \gt \left( \frac{n+1}{e}\right)^{n+1}$$dlatego$(n+1)\left( \frac{n}{e}\right)^n \left( \frac{n+1}{e}\right)^{-n-1}\gt 1$jest równoważne$\left(1+ \frac{1}{n}\right)^{n} \lt e$.

Na prawą stronę$$n! \lt \left(\frac{n+1}{2}\right)^{n} = e\left(\frac{n}{2}\right)^{n} \frac{\left(\frac{n+1}{2}\right)^{n}}{e\left(\frac{n}{2}\right)^{n}} = \\ =e\left(\frac{n}{2}\right)^{n} \frac{\left(1+ \frac{1}{n}\right)^{n}}{e} \lt e\left(\frac{n}{2}\right)^{n}$$

UNOwen Jan 25 2021 at 09:00

$$\displaystyle \frac{x\ln \left(x\right)}{\ln \left(x!\right)}=\frac{x\ln\left(x\right)}{\ln\left(\Gamma \left(x+1\right)\right)}$$

Stosując zasadę L'Hôpital,

$$\lim _{x\to \infty }\left(\frac{x\ln \left(x\right)}{\ln \left(\Gamma \:\left(x+1\right)\right)}\right)=\lim_{x\to \:\infty \:}\left(\displaystyle \frac{\ln(x)+1}{\psi \:^{\left(0\right)}\left(x+1\right)}\right)$$

Ponowne zastosowanie daje plony

$$\lim _{x\to \infty }\left(\frac{\frac{1}{x}}{\psi ^{\left(1\right)}\left(x+1\right)}\right)=\lim _{x\to \infty }\left(\frac{1}{x\left(\psi ^{\left(1\right)}\left(x+1\right)\right)}\right)$$

Mianownik zbliża się do 1 jako$x\rightarrow \infty$.